Finding KE of a rod with two particles and angular velocity?

In summary, finding the kinetic energy of a rod with two particles and angular velocity involves calculating the individual kinetic energy of each particle and adding them together, as well as taking into account the rotational kinetic energy due to the angular velocity of the rod. This can be done by using the formula for kinetic energy and considering the mass, velocity, and distance from the axis of rotation for each particle. The resulting total kinetic energy can then be used to analyze the motion and behavior of the system.
  • #1
rockchalk1312
38
0
In the figure, two particles, each with mass m = 0.79 kg, are fastened to each other, and to a rotation axis at O, by two thin rods, each with length d = 5.9 cm and mass M = 1.2 kg. The combination rotates around the rotation axis with angular speed ω = 0.32 rad/s. Measured about O, what is the combination's kinetic energy?I=Ʃmiri2
KE=1/2Iω2(.79kg)(.059m2)+(.79kg)(.118m2) = .0137 = I
1/2(.0137)(.32rad/s2) = 7.01E-4 = KE

Do I need to convert cm to m? Also how do you factor in the mass of the rod? I assume we need that, otherwise they wouldn't have given its mass, etc. I'm extremely new at all of this. Thank you!
 

Attachments

  • Screen Shot 2013-03-27 at 5.12.48 PM.png
    Screen Shot 2013-03-27 at 5.12.48 PM.png
    5.5 KB · Views: 814
Physics news on Phys.org
  • #2
Hello KU fan,

rockchalk1312 said:
Do I need to convert cm to m?

Well, yes, if you want to work in SI units (kilogram, meter, second units), you'll need to change the centimeters to meters. Doing so means the units of moment of inertia are the standard [kg·m2], and the kinetic energy is in units of Joules [kg·m2/s2].

You can work in some other system of units, but your final answer won't end up in units of Joules so easily.

Also how do you factor in the mass of the rod? I assume we need that, otherwise they wouldn't have given its mass, etc.
Yes, you do need that. You need to find the rods' contribution to the total moment of inertia.

Here is a useful link that might help (I often find it very helpful):
http://en.wikipedia.org/wiki/List_of_moments_of_inertia

rockchalk1312 said:
In the figure, two particles, each with mass m = 0.79 kg, are fastened to each other, and to a rotation axis at O, by two thin rods, each with length d = 5.9 cm and mass M = 1.2 kg. The combination rotates around the rotation axis with angular speed ω = 0.32 rad/s. Measured about O, what is the combination's kinetic energy?I=Ʃmiri2
KE=1/2Iω2(.79kg)(.059m2)+(.79kg)(.118m2) = .0137 = I
I think you missed the order of the squaring and the closing parenthesis when you typed in the numbers. I think you meant:

(.79 [kg])(.059 [m])2 + (.79 [kg])(.118 [m])2 = .0137 [kg·m2] = I

Anyway, besides that, you're missing something. Your approach would be correct if the rods were of negligible mass. But they're not. (The mass of each rod is heaver than the mass of each particle! The masses of the rods are definitely not negligible.)

You'll need to incorporate the contribution of the rods' moments of inertia into the total. :wink:
 
  • #3
collinsmark said:
Anyway, besides that, you're missing something. Your approach would be correct if the rods were of negligible mass. But they're not. (The mass of each rod is heaver than the mass of each particle! The masses of the rods are definitely not negligible.)

You'll need to incorporate the contribution of the rods' moments of inertia into the total. :wink:

Thank you for the reply!

So I went to your link and attempted to use the moment of inertia formula for the rod, I=mL2/3 which gave me (2.4kg)(.118m)2/3=.0111.

Then I added this to my original inertia for the two particles, .0111+.0137=.0248 and submitted this into the KE formula which gave me .00126 J and this was still the wrong answer.

Was it correct to use this formula and just double the mass and length for the whole rod?

Thank you again!
 
  • #4
rockchalk1312 said:
Thank you for the reply!

So I went to your link and attempted to use the moment of inertia formula for the rod, I=mL2/3 which gave me (2.4kg)(.118m)2/3=.0111.

Then I added this to my original inertia for the two particles, .0111+.0137=.0248

That's pretty much what I got (ignoring any minor rounding differences). :approve: (Don't forget your units though! :wink:)

and submitted this into the KE formula which gave me .00126 J

That's roughly what I got too (ignoring any minor rounding differences). :approve:

and this was still the wrong answer.

I'm not quite sure what to tell ya. Maybe it's the rounding errors that are the issue. You lost a little precision in your calculations. You might want to try again, but keep more significant figures in your intermediate calculations.

Was it correct to use this formula and just double the mass and length for the whole rod?

Yes, treating the two rods as one big rod is one valid way to solve this particular problem. (If the rods were not in a straight line, or were not connected at a point, it wouldn't work. But since both rods are behaving/configured as one, big, straight rod for this problem, it's okay to treat them as such.)

There is another way to do this problem too (and this is important for future, more complicated problems). You could treat both rods separately. The rod connected to the rotation axis has a moment of inertia of I=mL2/3. The other rod is a little more complicated though, but you can find its moment of inertia by using the parallel axis theorem.
 
Last edited:
  • #5


Yes, you will need to convert the length of the rods from centimeters to meters in order to get the correct units for the moment of inertia (I). This can be done by dividing the length by 100 (since there are 100 centimeters in 1 meter).

The mass of the rods is already factored into the moment of inertia equation, as it is represented by the term Ʃmiri2. This term takes into account the mass of each particle and its distance from the rotation axis.

To calculate the moment of inertia for the given system, you will need to add the individual moments of inertia for each particle and the two rods. So the equation would look like this:

I = (0.79 kg)(0.059 m)2 + (0.79 kg)(0.118 m)2 + (1.2 kg)(0.059 m)2 + (1.2 kg)(0.118 m)2

= 0.000137 kg*m2

Plugging this value into the kinetic energy equation, we get:

KE = 1/2 (0.000137 kg*m2)(0.32 rad/s)2

= 7.01E-4 J

So the kinetic energy of the system is approximately 0.0007 joules.
 

1. How do you determine the kinetic energy of a rod with two particles and angular velocity?

The kinetic energy (KE) of a rod with two particles and angular velocity can be determined by using the formula KE = (1/2)Iω^2, where I is the moment of inertia and ω is the angular velocity. The moment of inertia can be calculated by adding the individual moments of inertia of each particle, which is given by I = mr^2, where m is the mass and r is the distance from the axis of rotation.

2. What is the difference between linear and angular kinetic energy?

Linear kinetic energy is the energy possessed by an object due to its linear motion, while angular kinetic energy is the energy possessed by an object due to its rotational motion. Linear kinetic energy is calculated using the formula KE = (1/2)mv^2, where m is the mass and v is the linear velocity. Angular kinetic energy is calculated using the formula KE = (1/2)Iω^2, where I is the moment of inertia and ω is the angular velocity.

3. Can the kinetic energy of a rod with two particles and angular velocity be negative?

No, the kinetic energy of a system cannot be negative. It is always a positive value, as it represents the energy of motion possessed by the system. However, if the particles in the rod are moving in opposite directions, the total kinetic energy of the system can be zero.

4. How does the distance from the axis of rotation affect the kinetic energy of a rod with two particles and angular velocity?

The distance from the axis of rotation, also known as the moment arm, affects the moment of inertia and therefore the kinetic energy of the system. The farther the particles are from the axis of rotation, the larger the moment of inertia and the higher the kinetic energy. This is because the particles have to travel a greater distance in a given amount of time, resulting in a higher velocity and therefore a higher kinetic energy.

5. Can the kinetic energy of a rod with two particles and angular velocity be converted into potential energy?

Yes, the kinetic energy of a system can be converted into potential energy. This can occur if the particles in the rod are moving against a force, such as gravity. As the particles move against the force, they gain potential energy while losing kinetic energy. This conversion of energy is known as work and is described by the formula W = Fd, where F is the force and d is the displacement.

Similar threads

  • Introductory Physics Homework Help
Replies
10
Views
902
  • Introductory Physics Homework Help
Replies
5
Views
914
  • Introductory Physics Homework Help
2
Replies
38
Views
2K
  • Introductory Physics Homework Help
Replies
25
Views
1K
  • Introductory Physics Homework Help
Replies
2
Views
2K
  • Introductory Physics Homework Help
Replies
9
Views
836
  • Introductory Physics Homework Help
Replies
4
Views
826
  • Introductory Physics Homework Help
Replies
3
Views
1K
  • Introductory Physics Homework Help
Replies
25
Views
2K
  • Introductory Physics Homework Help
Replies
2
Views
1K
Back
Top